The expression x^2- x + 8.75 factors as (x + 2.5)(x - a) for some positive value "a". What is a?
step by step, please.

Answers

Answer 1

Answer:

3.5

Step-by-step explanation:

Write out quadratic

[tex] {x}^{2} - x + 8.75[/tex]

We are given that x+2.5 is a factor.

So our roots look like

[tex](x + 2.5)(x - a)[/tex]

Let apply the foil method to find some missing parts.

[tex]x \times x = {x}^{2} [/tex]

[tex]x \times - a = - ax[/tex]

[tex]2.5 \times x = 2.5[/tex]

[tex]2.5 \times - a = - 2.5a[/tex]

We know that the outer terms must multiply to 8.75. We also know that the middle terms must add up to -1.

[tex] - 2.5a = 8.75[/tex]

[tex] 2.5x - ax = -1[/tex]

The only number that satisfy this equation is

[tex] - 3.5[/tex]

Since it already negative, it is

[tex]3.5[/tex]


Related Questions

Solve for c........:

Answers

Mark my answer brilliant if it helps you.

plsssssssssssss help​

Answers

Answer:

negative linear relationship

Step-by-step explanation:

Which fraction is less?
2/5 or 1/6

Answers

Answer:

1/6

Step-by-step explanation:

convert ti decimal then compare the two

Answer:

Hello There!!

Step-by-step explanation:

1/6 is less.You can check this by 2÷5=0.4 and 1÷6=0.1666666667 so 1/6 is less.

hope this helps,have a great day!!

~Pinky~

Two consecutive even integers have a sum of 14. The equation that would be used to solve this
is x + x +__=14
The smallest of the two integers is _____ when you solve to find the integers.

Answers

x + x + 2 = 14
2x + 2 = 14
2x = 12, x = 6
Solution: the smallest of the two integers is 6
14 because xx stay as 0 just add 14

Pls help BRAINLYEST FOR BEST ANSWER YAYA

Answers

Answer:

A) 21

Step-by-step explanation:

Mean:

(23 + 18 + 24 + 15 + 25)/5

105/5

21

Answer:

21

The mean is 21. Remember to calculate the mean we add up the numbers then divide them by the number of numbers.

Which set of ordered pairs represents a function?
A {(22, 5), (23, 10), (22, 7), (23, 5)}
B {(22, 5), (26, 10), (23, 7), (23, 5)}
C {(22, 10), (23, 10), (24, 7), (25, 5)}
D {(24, 10), (23, 6), (22, 7), (24, 5)

Answers

Answer:

D

Step-by-step explanation:

the company is it a big company with you in my neighborhood that is a lot more expensive and more expensive to

Oil prices are projected to increase by 125% by next August. A quart of oil currently costs $1.25. What is the projected cost of a gallon of oil next August? Round your answer to the nearest hundredth.

Answers

Answer:$2.81

Step-by-step explanation:

1.25 is 100% of the price

125% + 100% = 225%

x = price by august

x = 225% (1.25)

225% = 2.25

2.25(1.25) = 2.8125 ≈ $2.81

Another way to see it:

=

225(1.25) / 100 = 2.8125

I WILL GIVE BRAINIEST IF ANSWERED WITH EXPLANATION PLEASE HELP
Find the area of each figure. Round your answer to 2 decimal places is required.

Answers

69.82 i think

Step-by-step explanation:

First the area of a rectangle is bh, so 5 times 12 is 60. Then you have a semicircle which is half a circle, so it would be pi(radius)squared divided by 2. The diameter is 5 because it's five on the other side and rectangles have opposite sides congruent. Radius is 2.5 after dividing diameter by 2. 2.5 pi r squared is like 19.634 blah blah blah. Divide that by two and add 60 to get 69.82.

What is the slope of the line through? (-4, 2) and (3, -3)?

Answers

Answer:

c. -5/7

Step-by-step explanation:

The amount of time, in hours, that a computer functions before breakingdown is a continuous random variable with probability density functiongiven byfX(x) ={1100e−x/100x≥00x <0.What is the probability that1. a computer will function between 50 and 150 hours before breakingdown (3 pts)2. It will function less than 100 hours? (3 pts)3. It will function exactly 100 hours before breaking down? (3 pts)

Answers

Answer:

[tex]P(50 < x < 150) =0.3834[/tex]

[tex]P(x = 100) =0.0074[/tex]

Step-by-step explanation:

Given

[tex]f(x) = \left \{ {{\frac{1}{100}e^{-x/100}\ x\ge 0} \atop {0\ x<0}} \right.[/tex]

Solving (a): Probability that it will function between 50 and 150 hr before it breaks down

This is represented as:

[tex]P(50 < x < 150) = \int\limits^{150}_{50} {f(x)} \, dx[/tex]

So, we have:

[tex]P(50 < x < 150) = \int\limits^{150}_{50} {\frac{1}{100}e^{-x/100}} \, dx[/tex]

Integrate:

[tex]P(50 < x < 150) =- e^{-x/100}|\limits^{150}_{50}[/tex]

This gives:

[tex]P(50 < x < 150) =- e^{-150/100} - - e^{-50/100}[/tex]

[tex]P(50 < x < 150) =- e^{-150/100} + e^{-50/100}[/tex]

[tex]P(50 < x < 150) =- e^{-1.5} + e^{-0.5}[/tex]

[tex]P(50 < x < 150) =- 0.2231 + 0.6065[/tex]

[tex]P(50 < x < 150) =0.3834[/tex]

Solving (a): Probability that it will function exactly 100 hr before it breaks down

This is represented as:

[tex]P(x= 100)[/tex]

This can be rewritten as:

[tex]P(x= 100) = P(99<x<101)[/tex]

So, we have:

[tex]P(99 < x < 101) = \int\limits^{101}_{99} {f(x)} \, dx[/tex]

So, we have:

[tex]P(99 < x < 101) = \int\limits^{101}_{99} {\frac{1}{100}e^{-x/100}} \, dx[/tex]

Integrate:

[tex]P(99 < x < 101) =- e^{-x/100}|\limits^{101}_{99}[/tex]

This gives:

[tex]P(99 < x < 101) =- e^{-101/100} - - e^{-99/100}[/tex]

[tex]P(99 < x < 101) =- e^{-101/100} + e^{-99/100}[/tex]

[tex]P(99 < x < 101) =- e^{-1.01} + e^{-0.99}[/tex]

[tex]P(99 < x < 101) =- 0.3642 + 0.3716[/tex]

[tex]P(99 < x < 101) =0.0074[/tex]

Hence:

[tex]P(x = 100) =P(99 < x < 101) =0.0074[/tex]

Pudding has 120 calories in 2 ounces. How many calories are in 8 ounces of the pudding?

Answers

Answer:

272 calories.

Step-by-step explanation:

jus solved it.

Answer:

480

Step-by-step explanation:

9) The area of the given rectangle is 40 square feet. Find the length and the width of the rectangle.
Length: x+1
Width: X + 4

Answers

Answer:

Step-by-step explanation:

hello : solve     (x+1)(x+4)=40

x²+5x+4 =40

x²+5x-36 =0      continu  us  delta

VERY EASY, WILL GIVE 50 POINTS FOR CORRECT ANSWER ASAP AND WILL GIVE BRAINLIEST.

Answers

Answer:

I would believe 90* clockwise.

Step-by-step explanation:

It rotated 90* to be located on its side.

HELP, ME PLEASE LOOK AT THE PHOTO TRUE OR FALSE

Answers

Answer:

The answer is false. 2 is not congruent to 5.

Honestly, someone has a different answer but they have a whole paragraph so I think you should listen to what they say.

Answer:

True

Step-by-step explanation:

Same-side exterior angle converse theorem states that if two lines are intersected by a transversal and the same-side exterior angle are congruent, then the two lines are parallel. One can apply this theorem to the given case, and thus lines A and B are parallel.

In ΔQRS, s = 2.3 inches, ∠S=51° and ∠Q=44°. Find the area of ΔQRS, to the nearest 10th of an square inch.

Answers

Answer:

Area of ΔQRS = 2.3 square inches

Step-by-step explanation:

From the given information,

<S + <Q + <R = [tex]180^{o}[/tex]

51 + 44 + <R = [tex]180^{o}[/tex]

95 + <R = [tex]180^{o}[/tex]

<R = [tex]180^{o}[/tex] - 95

    = [tex]85^{o}[/tex]

<R = [tex]85^{o}[/tex]

Applying the Sine rule, we have;

[tex]\frac{q}{SinQ}[/tex] = [tex]\frac{r}{SinR}[/tex] = [tex]\frac{s}{SinS}[/tex]

Using [tex]\frac{r}{SinR}[/tex] = [tex]\frac{s}{SinS}[/tex]

[tex]\frac{r}{Sin 85}[/tex] = [tex]\frac{2.3}{Sin51}[/tex]

r = [tex]\frac{2.3*Sin85}{sin51}[/tex]

  = 2.9483

r = 2.9 inches

Also, [tex]\frac{q}{SinQ}[/tex] = [tex]\frac{s}{SinS}[/tex]

[tex]\frac{q}{Sin44}[/tex] = [tex]\frac{2.3}{Sin51}[/tex]

q = [tex]\frac{2.3*Sin44}{Sin51}[/tex]

  = 2.0559

q = 2.0 inches

From Herons formula,

Area of a triangle = [tex]\sqrt{s(s-q)(s-r)(s-s)}[/tex]

s = [tex]\frac{2.3 + 2.0 + 2.9}{2}[/tex]

  = 3.6

Area of ΔQRS = [tex]\sqrt{3.6(3.6-2.0(3.6-2.9)(3.6-2.3)}[/tex]

                        = 2.2895

Area of ΔQRS = 2.3 square inches

Answer:

2.4

Step-by-step explanation:

Which expressions are equivalent?

1/5k-2/3j and -2/3j+1/5k

1/5k-2/3j and -1/5k+2/3j

1/5k-2/3j and 1/5j-2/3k

1/5k-2/3j and 2/3j-1/5k
please help!!!!!

Answers

The answer is 1/5k-2/3j and -2/3j+1/5k because it has the same values just rearranged.

What App or what Website can help me solve problems of Statistics? ​

Answers

the varsity tutors ap statisyics app (app)

URGENT WILL GIVE BRAINLIEST

Answers

Answer:

40

Step-by-step explanation:

100% of 80 is what number?
Select one:

160

80

8

100

Answers

Answer:

80

explanation:

100% means that its the whole of something so all of the said number

Answer:

In converting to percentage we multiply by 100%

100% of 80 =

80/100 x 100%

0.8 x 100 = 80

NB the 100% of any number is the number itself

find the image of U'( -8 , 1) after a translation along < 2 , 0 > and a reflection in the x axis
U"( -6 , -1)
U"(6 , 1)
U"(6 , -1)
U"(-1 , 6)

Answers

Given:

The point is U'(-8,1).

It is translated along <2,0> and reflected across the x-axis.

To find:

The image of U'.

Solution:

The given point is U'(-8,1).

It is translated along <2,0> . So, the rule of translation is:

[tex](x,y)\to (x+2,y+0)[/tex]

[tex]U'(-8,1)\to U_1(-8+2,1+0)[/tex]

[tex]U'(-8,1)\to U_1(-6,1)[/tex]

Then, it is reflected across the x-axis. So, the rule of reflection is:

[tex](x,y)\to (x,-y)[/tex]

[tex]U_1(-6,1)\to U''(-6,-1)[/tex]

The image of U'( -8 , 1) is U''(-6,-1).

Therefore, the correct option is A.

What’s 2/4 plus 3/8??
Mark brainliest!!!
No links!!

Answers

Step-by-step explanation:

[tex] \frac{2}{4} + \frac{3}{8} = \\ = \frac{4 + 3}{8} = \\ = \frac{7}{8} [/tex]

Answer:

The answer is 0.875 in decimal form but is 7/8 in fraction form

Step-by-step explanation:

How do I do this: 16 divided by 0.05

Answers

Answer:

320

Step-by-step explanation:

Step 1: Estimate the answer by rounding .

Step 2: If the divisor is not a whole number, then move the decimal place in places to the right to make it a whole number.

Step 3: Divide as usual.

Step 4: Put the decimal point in the quotient directly above where the decimal point now is in the dividend.

Answer:  320 is the correct answer

Step-by-step explanation:

To answer this questions is simple, you can see it as a long division question or regular division

Regular division:

16 ÷ 0.05 = 1600  

(You can transform this into a simple non-decimal question and solve easliy)

1600 ÷ 5 = 320

This is the way to solve these type of questions without straining your brain in the process.


what’s the answer ?

Answers

The answer will be:
247.75

Uh I can't explain well so heres the answer

247.75

Please helppp why is he right or wrong!!

Answers

Answer:

We cannot agree with Andre because x = 3.

Step-by-step explanation:

Assuming the diagram is a weighing scale which can also be represented as an equation having two sides that is balanced. We can express the situation as follows using equation:

x + 2 = 5

Solve for x

x + 2 - 2 = 5 - 2 (subtraction property of equality)

x = 3

If you plug in the value of x on the diagram given, the scale becomes balance on each side.

Therefore, x = 3. Andre is wrong and we cannot agree with Andre.

My friends and I go to the cinema by bus.
We spend 10 minutes walking and 15
minutes waiting at the bus stops, the
Journeys take 20 minutes each way
we arrive as the film starts. There are
25 minutes of adverts and trailers,
and the film lasts for 95 minutes
We get home at 17:20
What time did we set off to
the car​

Answers

Answer:

15:35??

Step-by-step explanation:

1. 20 + 15 = 35

2. 35 + 10= 45

3. 45 + 25 = 70 =  1 hour and 10 minutes

4. 95 mins = 1 hour and 45 minutes

5. 1 hour and 10 mins + 1 hour and 45 minutes = 2 hours and 55 minutes

6. 17:20

- 02:55

   15:35

15:35

i don't  know if this is right but it it does, your welcome.

helppppppppppppppppp

Answers

Answer: x > 4

x > 4 would look like that on the graph. If it was x is greater than or equal to 4, then the circle would be completely filled in. Hope this helped!

What type of world would you want to escape to right now if you could ?

Answers

I would want to escape to a world that I read about in one of my fantasy books. Life seems so much easier and simpler there. It’s too much violence and hatred in this world for me.

I attached the question please take a look

Answers

Answer:

e

Step-by-step explanation:

beishdhsjsvsudjdnbdhdjrjdxbxbudsndnbhhxhxbdhddbdbd

Can someone help me how to do this task please?

Answers

Answer:

11/12

Step-by-step explanation:

You’re answer is 11/12

h(a) = 3 - 2a ^ 2
h(- 3) =

Answers

Answer: -15

Step-by-step explanation:

First, calculate exponents.

-3 * -3 is 9.

Then multiply 2 and 9 which is 18.

3-18 is -15.

Other Questions
Which of the following inventories would a company ordinarily hold for sale? a. Work in process. b. Finished goods. c. Raw materials. d. Work in process and finished goods. On October 29, 2014, Lobo Co. Began operations by purchasing razors for resale. Lobo uses the perpetual inventory method. The razors have a 90-day warranty that requires the company to replace any nonworking razor. When a razor is returned, the company discards it and mails a new one from Merchandise Inventory to the customer. The company's cost per new razor is $15 and its retail selling price is $60 in both 2014 and 2015. The manufacturer has advised the company to expect warranty costs to equal 6% of dollar sales. The following transactions and events occurred. 2014 Nov. 11 Sold 70 razors for $4,200 cash. 30 Recognized warranty expense related to November sales with an adjusting entry. Dec. 9 Replaced 14 razors that were returned under the warranty. 16 Sold 210 razors for $12,600 cash. 29 Replaced 28 razors that were returned under the warranty. 31 Recognized warranty expense related to December sales with an adjusting entry. 2015 Jan. 5 Sold 140 razors for $8,400 cash. 17 Replaced 33 razors that were returned under the warranty. 31 Recognized warranty expense related to January sales with an adjusting entry. 1. 1 Prepare journal entries to record these transactions and adjustments for 2014. 1. 2 Prepare journal entries to record these transactions and adjustments for 2015 Let A = {1, 3, 5, 7}, B = {5, 6, 7, 8}, C = {5, 8}, D = {2, 5, 8}, and U={1, 2, 3, 4, 5, 6, 7, 8}. Use the sets above to find B U D. A. B U D = {5, 8} B. B U D = {6, 7} C. B U D = {2,5, 6, 7, 8} D. B U D = {1, 3, 4} E. None of the above The graduate class of the University of Flatland, which only graduates students with majors in mathematics, has 8 graduating seniors majoring in applied mathematics, 7 in statistics, and 6 in pure mathematics. What is the probability of choosing four of these graduates in such a way that they are of the same subdiscipline of mathematics? Find the exact interest for the following. Round to the nearest cent. A loan of $74,000 at 13% made on February 16 and due on June 30 O A. $3,580.78 OB $3,610.79 OC. $3,531,73 OD $3,660.94. earth's atmosphere blocks short wavelengths of the electromagnetic spectrum. which telescopes do not need to be placed in orbit around earth to observe short-length radiation? protocol to prevent unwanted network access and be configured to permit traffic from a specific address and provide security?a. WEP b. WPS c. MAC d. WPA 1 - A 60-day, 9% note for $40,800, dated May 1, is received from a customer on account. The maturity value of the note, assuming a 360-day year, isa.$40,800b.$612c.$44,4722 - If the individual subsidiary ledger accounts contained the following data:Cadence Company, Vendor, $100, credit balanceFranklin Enterprises, Customer, $334, debit balanceMarcelo Construction, Client, $442, debit balancePeyton Supplies, Supplier, $197, credit balanceThe accounts receivable (A/R) control account and the accounts payable (A/P) control account balances would be:a.A/R, $776; A/P, $297b.A/R, $297; A/P, $776c.A/R, $334; A/P, $442d.A/R, $197; A/P, $100 There has been much interest in whether the presence of 401(k) pension plans, availableto many U.S. workers, increases net savings. The data set 401ksubs.dta (available onBlackboard) contains information on net nancial assets (netfa), family income (inc), abinary variable for eligibility in a 401(k) plan (e401k), and several other variablesEstimate a linear probability model explaining 401(k) eligibility in terms of income,age, and gender. Include income and age in quadratic form. Explain your results. Use the following information to calculate Setans optimal bundle and his resulting utility:IC: U(x,y)= x^0.5 y^0.5Px=6,Py = 4, m=240 a 1 =4a, start subscript, 1, end subscript, equals, minus, 4 a_i = a_{i - 1} \cdot 2a i =a i1 2 A homeowner recorded the amount of electricity in kilowatt-hours (KWH) consumed in his house on each of 9 days. He also recorded the numbers of hours his air conditioner was turned on (AC). AC (hrs) 1.5 4.5 5.0 2.5 8.5 6.0 8.0 12.5 7.5 KWH 35 63 69 17 94 82 66 125 85 Use your calculator to answer the following question. Find the correlation between AC (hrs) and KWH. O-0.7567 0.8793 0.7941 0.9212 If f is a twice differentiable function and y is a function of x given by the parametric equationsY = f(t)AndX = t^2Thend^2y/dx^2 = Mention the type of analytical modeling relevant to each of the following points. (4 marks) 1. What changes will occur in the sales volume if the marketing department spends 20% less on electronic advertising? Please type your answer here:2. How will production be affected if 5 different machines are used? Please type your answer here:3. We should keep changing the machines until we can find a machine which can produce 100 units in 10 minutes. Please type your answer here:4. What is the most suitable advertising option that falls within our budget and feasibility? Please type your answer here: Co= So - PV(E) is the situation for an option to finish out of-the-money. O True O False Researchers wished to determine the size of a ice cream bowl that had an effect and how much a ice cream a person will add to their serving at an ice cream social people were randomly give. 17oz or 34oz bowls and then they served themselves Based on what you have learned in the chapter, about what percentage of all U.S families has little or no wealth?A) 10 percentB) 40 percentC) 70 percentD) 90 percent A coffee shop uses an online system to engage customers. After each transaction, customers receive a text asking them to rate their experience with either a smiley, neutral or sad face emoji. Specify the type of control that is illustrated in this case. prior to the 1948 presidential campaign, truman won back the support of labor unions by highlighting his opposition to the taft-hartley act. supporting the passage of the national security act. (q16) Jonathan is studying the income of people in state A. He finds out that the Lorenz curve for state A can be given as . Find the gini coefficient.